On uniform convergence Announcing the arrival of Valued Associate #679: Cesar Manara ...

NIntegrate on a solution of a matrix ODE

Vertical ranges of Column Plots in 12

draw a pulley system

Why not use the yoke to control yaw, as well as pitch and roll?

How do I say "this must not happen"?

How can I list files in reverse time order by a command and pass them as arguments to another command?

What is "Lambda" in Heston's original paper on stochastic volatility models?

Shimano 105 brifters (5800) and Avid BB5 compatibility

3D Masyu - A Die

Why did Bronn offer to be Tyrion Lannister's champion in trial by combat?

How to name indistinguishable henchmen in a screenplay?

As a dual citizen, my US passport will expire one day after traveling to the US. Will this work?

Why are two-digit numbers in Jonathan Swift's "Gulliver's Travels" (1726) written in "German style"?

By what mechanism was the 2017 UK General Election called?

Meaning of 境 in その日を境に

How does the body cool itself in a stillsuit?

What does Sonny Burch mean by, "S.H.I.E.L.D. and HYDRA don't even exist anymore"?

Any stored/leased 737s that could substitute for grounded MAXs?

Searching extreme points of polyhedron

Is a copyright notice with a non-existent name be invalid?

Why are current probes so expensive?

Where and when has Thucydides been studied?

One-one communication

Does a random sequence of vectors span a Hilbert space?



On uniform convergence



Announcing the arrival of Valued Associate #679: Cesar Manara
Planned maintenance scheduled April 23, 2019 at 23:30 UTC (7:30pm US/Eastern)A question on uniform convergenceUniform convergence of a sequence of functions?Uniform Convergence. Real AnalysisShowing uniform convergence of a sequence of functions which are inherited from a different functionClarity regarding a proof on uniform convergenceUniform convergence of indicatorsEquivalent formulation of uniform continuity (uniform convergence of shifted functions)Uniform convergence of productsUniform convergence and continuous functionUniform Convergence of $f_n^k$ and polynomial












3












$begingroup$


Let $f:mathbb{R}times [0,1]tomathbb{R}$ be a continuous function and ${x_n}$ a sequence of real numbers converging to $x$. Define



$g_n(y)=f(x_n,y)$, $0leq yleq1$,



$g(y)=f(x,y)$, $0leq yleq1$.



Show that $g_n$ converges to $g$ uniformly on $[0,1]$.



I don't need a complete solution, but a bit of direction would be extremely useful. Thank you!










share|cite|improve this question









$endgroup$












  • $begingroup$
    I would suggest using the fact that $[x-varepsilon,x+varepsilon]times[0,1]$ is compact for all $varepsilon>0$.
    $endgroup$
    – Floris Claassens
    Mar 26 at 11:26










  • $begingroup$
    @FlorisClaassens Yes, I was thinking of using compactness too. I have, in fact, given an answer here myself. Does that look okay?
    $endgroup$
    – WhySee
    Mar 26 at 11:41
















3












$begingroup$


Let $f:mathbb{R}times [0,1]tomathbb{R}$ be a continuous function and ${x_n}$ a sequence of real numbers converging to $x$. Define



$g_n(y)=f(x_n,y)$, $0leq yleq1$,



$g(y)=f(x,y)$, $0leq yleq1$.



Show that $g_n$ converges to $g$ uniformly on $[0,1]$.



I don't need a complete solution, but a bit of direction would be extremely useful. Thank you!










share|cite|improve this question









$endgroup$












  • $begingroup$
    I would suggest using the fact that $[x-varepsilon,x+varepsilon]times[0,1]$ is compact for all $varepsilon>0$.
    $endgroup$
    – Floris Claassens
    Mar 26 at 11:26










  • $begingroup$
    @FlorisClaassens Yes, I was thinking of using compactness too. I have, in fact, given an answer here myself. Does that look okay?
    $endgroup$
    – WhySee
    Mar 26 at 11:41














3












3








3





$begingroup$


Let $f:mathbb{R}times [0,1]tomathbb{R}$ be a continuous function and ${x_n}$ a sequence of real numbers converging to $x$. Define



$g_n(y)=f(x_n,y)$, $0leq yleq1$,



$g(y)=f(x,y)$, $0leq yleq1$.



Show that $g_n$ converges to $g$ uniformly on $[0,1]$.



I don't need a complete solution, but a bit of direction would be extremely useful. Thank you!










share|cite|improve this question









$endgroup$




Let $f:mathbb{R}times [0,1]tomathbb{R}$ be a continuous function and ${x_n}$ a sequence of real numbers converging to $x$. Define



$g_n(y)=f(x_n,y)$, $0leq yleq1$,



$g(y)=f(x,y)$, $0leq yleq1$.



Show that $g_n$ converges to $g$ uniformly on $[0,1]$.



I don't need a complete solution, but a bit of direction would be extremely useful. Thank you!







real-analysis uniform-convergence






share|cite|improve this question













share|cite|improve this question











share|cite|improve this question




share|cite|improve this question










asked Mar 26 at 11:16









WhySeeWhySee

36729




36729












  • $begingroup$
    I would suggest using the fact that $[x-varepsilon,x+varepsilon]times[0,1]$ is compact for all $varepsilon>0$.
    $endgroup$
    – Floris Claassens
    Mar 26 at 11:26










  • $begingroup$
    @FlorisClaassens Yes, I was thinking of using compactness too. I have, in fact, given an answer here myself. Does that look okay?
    $endgroup$
    – WhySee
    Mar 26 at 11:41


















  • $begingroup$
    I would suggest using the fact that $[x-varepsilon,x+varepsilon]times[0,1]$ is compact for all $varepsilon>0$.
    $endgroup$
    – Floris Claassens
    Mar 26 at 11:26










  • $begingroup$
    @FlorisClaassens Yes, I was thinking of using compactness too. I have, in fact, given an answer here myself. Does that look okay?
    $endgroup$
    – WhySee
    Mar 26 at 11:41
















$begingroup$
I would suggest using the fact that $[x-varepsilon,x+varepsilon]times[0,1]$ is compact for all $varepsilon>0$.
$endgroup$
– Floris Claassens
Mar 26 at 11:26




$begingroup$
I would suggest using the fact that $[x-varepsilon,x+varepsilon]times[0,1]$ is compact for all $varepsilon>0$.
$endgroup$
– Floris Claassens
Mar 26 at 11:26












$begingroup$
@FlorisClaassens Yes, I was thinking of using compactness too. I have, in fact, given an answer here myself. Does that look okay?
$endgroup$
– WhySee
Mar 26 at 11:41




$begingroup$
@FlorisClaassens Yes, I was thinking of using compactness too. I have, in fact, given an answer here myself. Does that look okay?
$endgroup$
– WhySee
Mar 26 at 11:41










4 Answers
4






active

oldest

votes


















3












$begingroup$

Let $K={x,x_1,x_2,cdots}$. Then $K$ is a compact set and $f$ is continuous, hence uniformly continuous on $K times [0,1]$. Now just write down the definition of uniform continuity and you will get the conclusion.






share|cite|improve this answer









$endgroup$





















    1












    $begingroup$

    Credit to Kavi.



    $f$ is uniformly continuos on $K={x,x_1,x_2,...}×[0,y],$ compact.



    $epsilon >0$ given, there exists a $delta >0$ s.t.



    $||(x,'y')-(x,y)|| lt delta$ implies



    $|f(x',y')-f(x,y)| lt epsilon$.



    With $y'=y$:



    $||(x',y)-(x,y)|| lt delta$ implies



    $|f(x',y)-f(x,y)| lt epsilon$, i.e.



    $|x'-x| lt delta$ implies



    $|f(x',y)-f(x,y)| lt epsilon$.



    Since $x_n$ converges to $x$:



    For a $delta >0$ there is a $n_0$ s.t.



    for $n ge n_0$ :



    We have $|x_n -x| lt delta$ which implies



    $|f(x_n,y)-f(x,y)| lt epsilon$.






    share|cite|improve this answer









    $endgroup$





















      0












      $begingroup$

      Let $epsilon>0$ be given.



      Now, $left|g_n(y)-g(y)right|=left|f(x_n,y)-f(x,y)right|$

      As $f$ is continuous, therefore for each $yinleft[0,1right]$, $exists N_{epsilon,y}inmathbb{N}$ such that $left|f(x_n,y)-f(x,y)right|<epsilon$ whenever $ngeq N_{epsilon,y}$.



      Now, $left[0,1right]=bigcup_{yinleft[0,1right]}(y-epsilon,y+epsilon)$. As $left[0,1right]$ is compact, there exists a finite subcover. Thus, there exists $minmathbb{N}$ such that $left[0,1right]=bigcuplimits_{i=1}^{m}(y_i-epsilon,y_i+epsilon)$. Now any $yinleft[0,1right]$ belongs to one of the intervals $(y_i-epsilon,y_i+epsilon)$ for some $i$. Thus, taking $N=$ max${N_{epsilon,y_i}: i=1,2,...,m}$, we get that $g_n$ converges to $g$ uniformly on $left[0,1right]$.






      share|cite|improve this answer









      $endgroup$









      • 1




        $begingroup$
        That does not work as there is no guarantee that $|f(x_{n},y)-f(x_{n},y_{i}|<varepsilon$ for any $yin(y_{i}-varepsilon,y_{i}+varepsilon)$. I'd suggest the following: As $f$ is continuous for all $y$ there exists a $delta_{y}>0$ such that for all $(a,b)in(x-delta,x+delta)times(y-delta,y+delta)$ we have $|f(x,y)-f(a,b)|<varepsilon$. You can then use the fact that ${x}times[0,1]$ is compact. (This time I actually worked out a proper solution, so I'm certain this approach will work.
        $endgroup$
        – Floris Claassens
        Mar 26 at 11:53



















      0












      $begingroup$

      Here's a certainly much cleaner way (tbh, I didn't check your solution properly):



      We want to show that $g_n$ converges uniformly to $g$, which is the same as asking that $$lim_{n to + infty} sup_{y in [0,1]} vert g_n(y)-g(y) vert =0.$$ Now observe that $vert g_n(y)-g(y)vert$ is a continuous function on $[0,1]$ for each $n$, hence it attains its maximum. Therefore denote $y_n in [0,1]$ such that $$sup_{y in [0,1]} vert g_n(y)-g(y) vert = vert g_n(y_n)-g(y_n) vert.$$ At the same time we have $$vert g_n(y_n)-g(y_n) vert=vert f(x_n,y_n)-f(x,y_n) vert.$$ Since $[0,1]$ is compact, we can pick a subsequence $y_{n_k}$ such that $y_{n_k}$ converges to some $y$ and as $x_n$ already converges to $x$ and so will its corresponding subsequence $x_{n_k}$.



      Using that $f$ is continuous in both entries now gives the desired claim:



      $$lim_{n to infty}sup_{y in [0,1]}vert g_n(y)-g(y) vert=lim_{k to infty} vert f(x_{n_k},y_{n_k})-f(x,y_{n_k})vert=0. $$






      share|cite|improve this answer









      $endgroup$














        Your Answer








        StackExchange.ready(function() {
        var channelOptions = {
        tags: "".split(" "),
        id: "69"
        };
        initTagRenderer("".split(" "), "".split(" "), channelOptions);

        StackExchange.using("externalEditor", function() {
        // Have to fire editor after snippets, if snippets enabled
        if (StackExchange.settings.snippets.snippetsEnabled) {
        StackExchange.using("snippets", function() {
        createEditor();
        });
        }
        else {
        createEditor();
        }
        });

        function createEditor() {
        StackExchange.prepareEditor({
        heartbeatType: 'answer',
        autoActivateHeartbeat: false,
        convertImagesToLinks: true,
        noModals: true,
        showLowRepImageUploadWarning: true,
        reputationToPostImages: 10,
        bindNavPrevention: true,
        postfix: "",
        imageUploader: {
        brandingHtml: "Powered by u003ca class="icon-imgur-white" href="https://imgur.com/"u003eu003c/au003e",
        contentPolicyHtml: "User contributions licensed under u003ca href="https://creativecommons.org/licenses/by-sa/3.0/"u003ecc by-sa 3.0 with attribution requiredu003c/au003e u003ca href="https://stackoverflow.com/legal/content-policy"u003e(content policy)u003c/au003e",
        allowUrls: true
        },
        noCode: true, onDemand: true,
        discardSelector: ".discard-answer"
        ,immediatelyShowMarkdownHelp:true
        });


        }
        });














        draft saved

        draft discarded


















        StackExchange.ready(
        function () {
        StackExchange.openid.initPostLogin('.new-post-login', 'https%3a%2f%2fmath.stackexchange.com%2fquestions%2f3163041%2fon-uniform-convergence%23new-answer', 'question_page');
        }
        );

        Post as a guest















        Required, but never shown

























        4 Answers
        4






        active

        oldest

        votes








        4 Answers
        4






        active

        oldest

        votes









        active

        oldest

        votes






        active

        oldest

        votes









        3












        $begingroup$

        Let $K={x,x_1,x_2,cdots}$. Then $K$ is a compact set and $f$ is continuous, hence uniformly continuous on $K times [0,1]$. Now just write down the definition of uniform continuity and you will get the conclusion.






        share|cite|improve this answer









        $endgroup$


















          3












          $begingroup$

          Let $K={x,x_1,x_2,cdots}$. Then $K$ is a compact set and $f$ is continuous, hence uniformly continuous on $K times [0,1]$. Now just write down the definition of uniform continuity and you will get the conclusion.






          share|cite|improve this answer









          $endgroup$
















            3












            3








            3





            $begingroup$

            Let $K={x,x_1,x_2,cdots}$. Then $K$ is a compact set and $f$ is continuous, hence uniformly continuous on $K times [0,1]$. Now just write down the definition of uniform continuity and you will get the conclusion.






            share|cite|improve this answer









            $endgroup$



            Let $K={x,x_1,x_2,cdots}$. Then $K$ is a compact set and $f$ is continuous, hence uniformly continuous on $K times [0,1]$. Now just write down the definition of uniform continuity and you will get the conclusion.







            share|cite|improve this answer












            share|cite|improve this answer



            share|cite|improve this answer










            answered Mar 26 at 12:19









            Kavi Rama MurthyKavi Rama Murthy

            76.6k53471




            76.6k53471























                1












                $begingroup$

                Credit to Kavi.



                $f$ is uniformly continuos on $K={x,x_1,x_2,...}×[0,y],$ compact.



                $epsilon >0$ given, there exists a $delta >0$ s.t.



                $||(x,'y')-(x,y)|| lt delta$ implies



                $|f(x',y')-f(x,y)| lt epsilon$.



                With $y'=y$:



                $||(x',y)-(x,y)|| lt delta$ implies



                $|f(x',y)-f(x,y)| lt epsilon$, i.e.



                $|x'-x| lt delta$ implies



                $|f(x',y)-f(x,y)| lt epsilon$.



                Since $x_n$ converges to $x$:



                For a $delta >0$ there is a $n_0$ s.t.



                for $n ge n_0$ :



                We have $|x_n -x| lt delta$ which implies



                $|f(x_n,y)-f(x,y)| lt epsilon$.






                share|cite|improve this answer









                $endgroup$


















                  1












                  $begingroup$

                  Credit to Kavi.



                  $f$ is uniformly continuos on $K={x,x_1,x_2,...}×[0,y],$ compact.



                  $epsilon >0$ given, there exists a $delta >0$ s.t.



                  $||(x,'y')-(x,y)|| lt delta$ implies



                  $|f(x',y')-f(x,y)| lt epsilon$.



                  With $y'=y$:



                  $||(x',y)-(x,y)|| lt delta$ implies



                  $|f(x',y)-f(x,y)| lt epsilon$, i.e.



                  $|x'-x| lt delta$ implies



                  $|f(x',y)-f(x,y)| lt epsilon$.



                  Since $x_n$ converges to $x$:



                  For a $delta >0$ there is a $n_0$ s.t.



                  for $n ge n_0$ :



                  We have $|x_n -x| lt delta$ which implies



                  $|f(x_n,y)-f(x,y)| lt epsilon$.






                  share|cite|improve this answer









                  $endgroup$
















                    1












                    1








                    1





                    $begingroup$

                    Credit to Kavi.



                    $f$ is uniformly continuos on $K={x,x_1,x_2,...}×[0,y],$ compact.



                    $epsilon >0$ given, there exists a $delta >0$ s.t.



                    $||(x,'y')-(x,y)|| lt delta$ implies



                    $|f(x',y')-f(x,y)| lt epsilon$.



                    With $y'=y$:



                    $||(x',y)-(x,y)|| lt delta$ implies



                    $|f(x',y)-f(x,y)| lt epsilon$, i.e.



                    $|x'-x| lt delta$ implies



                    $|f(x',y)-f(x,y)| lt epsilon$.



                    Since $x_n$ converges to $x$:



                    For a $delta >0$ there is a $n_0$ s.t.



                    for $n ge n_0$ :



                    We have $|x_n -x| lt delta$ which implies



                    $|f(x_n,y)-f(x,y)| lt epsilon$.






                    share|cite|improve this answer









                    $endgroup$



                    Credit to Kavi.



                    $f$ is uniformly continuos on $K={x,x_1,x_2,...}×[0,y],$ compact.



                    $epsilon >0$ given, there exists a $delta >0$ s.t.



                    $||(x,'y')-(x,y)|| lt delta$ implies



                    $|f(x',y')-f(x,y)| lt epsilon$.



                    With $y'=y$:



                    $||(x',y)-(x,y)|| lt delta$ implies



                    $|f(x',y)-f(x,y)| lt epsilon$, i.e.



                    $|x'-x| lt delta$ implies



                    $|f(x',y)-f(x,y)| lt epsilon$.



                    Since $x_n$ converges to $x$:



                    For a $delta >0$ there is a $n_0$ s.t.



                    for $n ge n_0$ :



                    We have $|x_n -x| lt delta$ which implies



                    $|f(x_n,y)-f(x,y)| lt epsilon$.







                    share|cite|improve this answer












                    share|cite|improve this answer



                    share|cite|improve this answer










                    answered Mar 26 at 19:27









                    Peter SzilasPeter Szilas

                    12k2822




                    12k2822























                        0












                        $begingroup$

                        Let $epsilon>0$ be given.



                        Now, $left|g_n(y)-g(y)right|=left|f(x_n,y)-f(x,y)right|$

                        As $f$ is continuous, therefore for each $yinleft[0,1right]$, $exists N_{epsilon,y}inmathbb{N}$ such that $left|f(x_n,y)-f(x,y)right|<epsilon$ whenever $ngeq N_{epsilon,y}$.



                        Now, $left[0,1right]=bigcup_{yinleft[0,1right]}(y-epsilon,y+epsilon)$. As $left[0,1right]$ is compact, there exists a finite subcover. Thus, there exists $minmathbb{N}$ such that $left[0,1right]=bigcuplimits_{i=1}^{m}(y_i-epsilon,y_i+epsilon)$. Now any $yinleft[0,1right]$ belongs to one of the intervals $(y_i-epsilon,y_i+epsilon)$ for some $i$. Thus, taking $N=$ max${N_{epsilon,y_i}: i=1,2,...,m}$, we get that $g_n$ converges to $g$ uniformly on $left[0,1right]$.






                        share|cite|improve this answer









                        $endgroup$









                        • 1




                          $begingroup$
                          That does not work as there is no guarantee that $|f(x_{n},y)-f(x_{n},y_{i}|<varepsilon$ for any $yin(y_{i}-varepsilon,y_{i}+varepsilon)$. I'd suggest the following: As $f$ is continuous for all $y$ there exists a $delta_{y}>0$ such that for all $(a,b)in(x-delta,x+delta)times(y-delta,y+delta)$ we have $|f(x,y)-f(a,b)|<varepsilon$. You can then use the fact that ${x}times[0,1]$ is compact. (This time I actually worked out a proper solution, so I'm certain this approach will work.
                          $endgroup$
                          – Floris Claassens
                          Mar 26 at 11:53
















                        0












                        $begingroup$

                        Let $epsilon>0$ be given.



                        Now, $left|g_n(y)-g(y)right|=left|f(x_n,y)-f(x,y)right|$

                        As $f$ is continuous, therefore for each $yinleft[0,1right]$, $exists N_{epsilon,y}inmathbb{N}$ such that $left|f(x_n,y)-f(x,y)right|<epsilon$ whenever $ngeq N_{epsilon,y}$.



                        Now, $left[0,1right]=bigcup_{yinleft[0,1right]}(y-epsilon,y+epsilon)$. As $left[0,1right]$ is compact, there exists a finite subcover. Thus, there exists $minmathbb{N}$ such that $left[0,1right]=bigcuplimits_{i=1}^{m}(y_i-epsilon,y_i+epsilon)$. Now any $yinleft[0,1right]$ belongs to one of the intervals $(y_i-epsilon,y_i+epsilon)$ for some $i$. Thus, taking $N=$ max${N_{epsilon,y_i}: i=1,2,...,m}$, we get that $g_n$ converges to $g$ uniformly on $left[0,1right]$.






                        share|cite|improve this answer









                        $endgroup$









                        • 1




                          $begingroup$
                          That does not work as there is no guarantee that $|f(x_{n},y)-f(x_{n},y_{i}|<varepsilon$ for any $yin(y_{i}-varepsilon,y_{i}+varepsilon)$. I'd suggest the following: As $f$ is continuous for all $y$ there exists a $delta_{y}>0$ such that for all $(a,b)in(x-delta,x+delta)times(y-delta,y+delta)$ we have $|f(x,y)-f(a,b)|<varepsilon$. You can then use the fact that ${x}times[0,1]$ is compact. (This time I actually worked out a proper solution, so I'm certain this approach will work.
                          $endgroup$
                          – Floris Claassens
                          Mar 26 at 11:53














                        0












                        0








                        0





                        $begingroup$

                        Let $epsilon>0$ be given.



                        Now, $left|g_n(y)-g(y)right|=left|f(x_n,y)-f(x,y)right|$

                        As $f$ is continuous, therefore for each $yinleft[0,1right]$, $exists N_{epsilon,y}inmathbb{N}$ such that $left|f(x_n,y)-f(x,y)right|<epsilon$ whenever $ngeq N_{epsilon,y}$.



                        Now, $left[0,1right]=bigcup_{yinleft[0,1right]}(y-epsilon,y+epsilon)$. As $left[0,1right]$ is compact, there exists a finite subcover. Thus, there exists $minmathbb{N}$ such that $left[0,1right]=bigcuplimits_{i=1}^{m}(y_i-epsilon,y_i+epsilon)$. Now any $yinleft[0,1right]$ belongs to one of the intervals $(y_i-epsilon,y_i+epsilon)$ for some $i$. Thus, taking $N=$ max${N_{epsilon,y_i}: i=1,2,...,m}$, we get that $g_n$ converges to $g$ uniformly on $left[0,1right]$.






                        share|cite|improve this answer









                        $endgroup$



                        Let $epsilon>0$ be given.



                        Now, $left|g_n(y)-g(y)right|=left|f(x_n,y)-f(x,y)right|$

                        As $f$ is continuous, therefore for each $yinleft[0,1right]$, $exists N_{epsilon,y}inmathbb{N}$ such that $left|f(x_n,y)-f(x,y)right|<epsilon$ whenever $ngeq N_{epsilon,y}$.



                        Now, $left[0,1right]=bigcup_{yinleft[0,1right]}(y-epsilon,y+epsilon)$. As $left[0,1right]$ is compact, there exists a finite subcover. Thus, there exists $minmathbb{N}$ such that $left[0,1right]=bigcuplimits_{i=1}^{m}(y_i-epsilon,y_i+epsilon)$. Now any $yinleft[0,1right]$ belongs to one of the intervals $(y_i-epsilon,y_i+epsilon)$ for some $i$. Thus, taking $N=$ max${N_{epsilon,y_i}: i=1,2,...,m}$, we get that $g_n$ converges to $g$ uniformly on $left[0,1right]$.







                        share|cite|improve this answer












                        share|cite|improve this answer



                        share|cite|improve this answer










                        answered Mar 26 at 11:40









                        WhySeeWhySee

                        36729




                        36729








                        • 1




                          $begingroup$
                          That does not work as there is no guarantee that $|f(x_{n},y)-f(x_{n},y_{i}|<varepsilon$ for any $yin(y_{i}-varepsilon,y_{i}+varepsilon)$. I'd suggest the following: As $f$ is continuous for all $y$ there exists a $delta_{y}>0$ such that for all $(a,b)in(x-delta,x+delta)times(y-delta,y+delta)$ we have $|f(x,y)-f(a,b)|<varepsilon$. You can then use the fact that ${x}times[0,1]$ is compact. (This time I actually worked out a proper solution, so I'm certain this approach will work.
                          $endgroup$
                          – Floris Claassens
                          Mar 26 at 11:53














                        • 1




                          $begingroup$
                          That does not work as there is no guarantee that $|f(x_{n},y)-f(x_{n},y_{i}|<varepsilon$ for any $yin(y_{i}-varepsilon,y_{i}+varepsilon)$. I'd suggest the following: As $f$ is continuous for all $y$ there exists a $delta_{y}>0$ such that for all $(a,b)in(x-delta,x+delta)times(y-delta,y+delta)$ we have $|f(x,y)-f(a,b)|<varepsilon$. You can then use the fact that ${x}times[0,1]$ is compact. (This time I actually worked out a proper solution, so I'm certain this approach will work.
                          $endgroup$
                          – Floris Claassens
                          Mar 26 at 11:53








                        1




                        1




                        $begingroup$
                        That does not work as there is no guarantee that $|f(x_{n},y)-f(x_{n},y_{i}|<varepsilon$ for any $yin(y_{i}-varepsilon,y_{i}+varepsilon)$. I'd suggest the following: As $f$ is continuous for all $y$ there exists a $delta_{y}>0$ such that for all $(a,b)in(x-delta,x+delta)times(y-delta,y+delta)$ we have $|f(x,y)-f(a,b)|<varepsilon$. You can then use the fact that ${x}times[0,1]$ is compact. (This time I actually worked out a proper solution, so I'm certain this approach will work.
                        $endgroup$
                        – Floris Claassens
                        Mar 26 at 11:53




                        $begingroup$
                        That does not work as there is no guarantee that $|f(x_{n},y)-f(x_{n},y_{i}|<varepsilon$ for any $yin(y_{i}-varepsilon,y_{i}+varepsilon)$. I'd suggest the following: As $f$ is continuous for all $y$ there exists a $delta_{y}>0$ such that for all $(a,b)in(x-delta,x+delta)times(y-delta,y+delta)$ we have $|f(x,y)-f(a,b)|<varepsilon$. You can then use the fact that ${x}times[0,1]$ is compact. (This time I actually worked out a proper solution, so I'm certain this approach will work.
                        $endgroup$
                        – Floris Claassens
                        Mar 26 at 11:53











                        0












                        $begingroup$

                        Here's a certainly much cleaner way (tbh, I didn't check your solution properly):



                        We want to show that $g_n$ converges uniformly to $g$, which is the same as asking that $$lim_{n to + infty} sup_{y in [0,1]} vert g_n(y)-g(y) vert =0.$$ Now observe that $vert g_n(y)-g(y)vert$ is a continuous function on $[0,1]$ for each $n$, hence it attains its maximum. Therefore denote $y_n in [0,1]$ such that $$sup_{y in [0,1]} vert g_n(y)-g(y) vert = vert g_n(y_n)-g(y_n) vert.$$ At the same time we have $$vert g_n(y_n)-g(y_n) vert=vert f(x_n,y_n)-f(x,y_n) vert.$$ Since $[0,1]$ is compact, we can pick a subsequence $y_{n_k}$ such that $y_{n_k}$ converges to some $y$ and as $x_n$ already converges to $x$ and so will its corresponding subsequence $x_{n_k}$.



                        Using that $f$ is continuous in both entries now gives the desired claim:



                        $$lim_{n to infty}sup_{y in [0,1]}vert g_n(y)-g(y) vert=lim_{k to infty} vert f(x_{n_k},y_{n_k})-f(x,y_{n_k})vert=0. $$






                        share|cite|improve this answer









                        $endgroup$


















                          0












                          $begingroup$

                          Here's a certainly much cleaner way (tbh, I didn't check your solution properly):



                          We want to show that $g_n$ converges uniformly to $g$, which is the same as asking that $$lim_{n to + infty} sup_{y in [0,1]} vert g_n(y)-g(y) vert =0.$$ Now observe that $vert g_n(y)-g(y)vert$ is a continuous function on $[0,1]$ for each $n$, hence it attains its maximum. Therefore denote $y_n in [0,1]$ such that $$sup_{y in [0,1]} vert g_n(y)-g(y) vert = vert g_n(y_n)-g(y_n) vert.$$ At the same time we have $$vert g_n(y_n)-g(y_n) vert=vert f(x_n,y_n)-f(x,y_n) vert.$$ Since $[0,1]$ is compact, we can pick a subsequence $y_{n_k}$ such that $y_{n_k}$ converges to some $y$ and as $x_n$ already converges to $x$ and so will its corresponding subsequence $x_{n_k}$.



                          Using that $f$ is continuous in both entries now gives the desired claim:



                          $$lim_{n to infty}sup_{y in [0,1]}vert g_n(y)-g(y) vert=lim_{k to infty} vert f(x_{n_k},y_{n_k})-f(x,y_{n_k})vert=0. $$






                          share|cite|improve this answer









                          $endgroup$
















                            0












                            0








                            0





                            $begingroup$

                            Here's a certainly much cleaner way (tbh, I didn't check your solution properly):



                            We want to show that $g_n$ converges uniformly to $g$, which is the same as asking that $$lim_{n to + infty} sup_{y in [0,1]} vert g_n(y)-g(y) vert =0.$$ Now observe that $vert g_n(y)-g(y)vert$ is a continuous function on $[0,1]$ for each $n$, hence it attains its maximum. Therefore denote $y_n in [0,1]$ such that $$sup_{y in [0,1]} vert g_n(y)-g(y) vert = vert g_n(y_n)-g(y_n) vert.$$ At the same time we have $$vert g_n(y_n)-g(y_n) vert=vert f(x_n,y_n)-f(x,y_n) vert.$$ Since $[0,1]$ is compact, we can pick a subsequence $y_{n_k}$ such that $y_{n_k}$ converges to some $y$ and as $x_n$ already converges to $x$ and so will its corresponding subsequence $x_{n_k}$.



                            Using that $f$ is continuous in both entries now gives the desired claim:



                            $$lim_{n to infty}sup_{y in [0,1]}vert g_n(y)-g(y) vert=lim_{k to infty} vert f(x_{n_k},y_{n_k})-f(x,y_{n_k})vert=0. $$






                            share|cite|improve this answer









                            $endgroup$



                            Here's a certainly much cleaner way (tbh, I didn't check your solution properly):



                            We want to show that $g_n$ converges uniformly to $g$, which is the same as asking that $$lim_{n to + infty} sup_{y in [0,1]} vert g_n(y)-g(y) vert =0.$$ Now observe that $vert g_n(y)-g(y)vert$ is a continuous function on $[0,1]$ for each $n$, hence it attains its maximum. Therefore denote $y_n in [0,1]$ such that $$sup_{y in [0,1]} vert g_n(y)-g(y) vert = vert g_n(y_n)-g(y_n) vert.$$ At the same time we have $$vert g_n(y_n)-g(y_n) vert=vert f(x_n,y_n)-f(x,y_n) vert.$$ Since $[0,1]$ is compact, we can pick a subsequence $y_{n_k}$ such that $y_{n_k}$ converges to some $y$ and as $x_n$ already converges to $x$ and so will its corresponding subsequence $x_{n_k}$.



                            Using that $f$ is continuous in both entries now gives the desired claim:



                            $$lim_{n to infty}sup_{y in [0,1]}vert g_n(y)-g(y) vert=lim_{k to infty} vert f(x_{n_k},y_{n_k})-f(x,y_{n_k})vert=0. $$







                            share|cite|improve this answer












                            share|cite|improve this answer



                            share|cite|improve this answer










                            answered Mar 26 at 11:57









                            noctusraidnoctusraid

                            8791727




                            8791727






























                                draft saved

                                draft discarded




















































                                Thanks for contributing an answer to Mathematics Stack Exchange!


                                • Please be sure to answer the question. Provide details and share your research!

                                But avoid



                                • Asking for help, clarification, or responding to other answers.

                                • Making statements based on opinion; back them up with references or personal experience.


                                Use MathJax to format equations. MathJax reference.


                                To learn more, see our tips on writing great answers.




                                draft saved


                                draft discarded














                                StackExchange.ready(
                                function () {
                                StackExchange.openid.initPostLogin('.new-post-login', 'https%3a%2f%2fmath.stackexchange.com%2fquestions%2f3163041%2fon-uniform-convergence%23new-answer', 'question_page');
                                }
                                );

                                Post as a guest















                                Required, but never shown





















































                                Required, but never shown














                                Required, but never shown












                                Required, but never shown







                                Required, but never shown

































                                Required, but never shown














                                Required, but never shown












                                Required, but never shown







                                Required, but never shown







                                Popular posts from this blog

                                Magento 2 - Add success message with knockout Planned maintenance scheduled April 23, 2019 at 23:30 UTC (7:30pm US/Eastern) Announcing the arrival of Valued Associate #679: Cesar Manara Unicorn Meta Zoo #1: Why another podcast?Success / Error message on ajax request$.widget is not a function when loading a homepage after add custom jQuery on custom themeHow can bind jQuery to current document in Magento 2 When template load by ajaxRedirect page using plugin in Magento 2Magento 2 - Update quantity and totals of cart page without page reload?Magento 2: Quote data not loaded on knockout checkoutMagento 2 : I need to change add to cart success message after adding product into cart through pluginMagento 2.2.5 How to add additional products to cart from new checkout step?Magento 2 Add error/success message with knockoutCan't validate Post Code on checkout page

                                Fil:Tokke komm.svg

                                Where did Arya get these scars? Unicorn Meta Zoo #1: Why another podcast? Announcing the arrival of Valued Associate #679: Cesar Manara Favourite questions and answers from the 1st quarter of 2019Why did Arya refuse to end it?Has the pronunciation of Arya Stark's name changed?Has Arya forgiven people?Why did Arya Stark lose her vision?Why can Arya still use the faces?Has the Narrow Sea become narrower?Does Arya Stark know how to make poisons outside of the House of Black and White?Why did Nymeria leave Arya?Why did Arya not kill the Lannister soldiers she encountered in the Riverlands?What is the current canonical age of Sansa, Bran and Arya Stark?